Knights and Knaves question












4












$begingroup$


You are on a land inhabited by Knights and Knaves. Knights will always tell the truth and knaves always lie.



You meet three inhabitants(A, B, and C), and ask how many of them are knaves. A answers so quietly, so you ask B what A had said. B says that A had said that exactly two of them were knaves. C says B is lying.



Is it possible to know what A is?
Further, what are B and C.










share|improve this question









New contributor




user58804 is a new contributor to this site. Take care in asking for clarification, commenting, and answering.
Check out our Code of Conduct.







$endgroup$








  • 1




    $begingroup$
    Please don't change your question after someone has answered it. Now the answer references D, E, and F, and makes no sense. I'm editing the answer to match the revised question, but in the future it's best not to make changes that make the answers obsolete ... especially when the changes are superficial like this.
    $endgroup$
    – Rubio
    1 hour ago
















4












$begingroup$


You are on a land inhabited by Knights and Knaves. Knights will always tell the truth and knaves always lie.



You meet three inhabitants(A, B, and C), and ask how many of them are knaves. A answers so quietly, so you ask B what A had said. B says that A had said that exactly two of them were knaves. C says B is lying.



Is it possible to know what A is?
Further, what are B and C.










share|improve this question









New contributor




user58804 is a new contributor to this site. Take care in asking for clarification, commenting, and answering.
Check out our Code of Conduct.







$endgroup$








  • 1




    $begingroup$
    Please don't change your question after someone has answered it. Now the answer references D, E, and F, and makes no sense. I'm editing the answer to match the revised question, but in the future it's best not to make changes that make the answers obsolete ... especially when the changes are superficial like this.
    $endgroup$
    – Rubio
    1 hour ago














4












4








4





$begingroup$


You are on a land inhabited by Knights and Knaves. Knights will always tell the truth and knaves always lie.



You meet three inhabitants(A, B, and C), and ask how many of them are knaves. A answers so quietly, so you ask B what A had said. B says that A had said that exactly two of them were knaves. C says B is lying.



Is it possible to know what A is?
Further, what are B and C.










share|improve this question









New contributor




user58804 is a new contributor to this site. Take care in asking for clarification, commenting, and answering.
Check out our Code of Conduct.







$endgroup$




You are on a land inhabited by Knights and Knaves. Knights will always tell the truth and knaves always lie.



You meet three inhabitants(A, B, and C), and ask how many of them are knaves. A answers so quietly, so you ask B what A had said. B says that A had said that exactly two of them were knaves. C says B is lying.



Is it possible to know what A is?
Further, what are B and C.







logical-deduction liars






share|improve this question









New contributor




user58804 is a new contributor to this site. Take care in asking for clarification, commenting, and answering.
Check out our Code of Conduct.











share|improve this question









New contributor




user58804 is a new contributor to this site. Take care in asking for clarification, commenting, and answering.
Check out our Code of Conduct.









share|improve this question




share|improve this question








edited 2 hours ago







user58804













New contributor




user58804 is a new contributor to this site. Take care in asking for clarification, commenting, and answering.
Check out our Code of Conduct.









asked 3 hours ago









user58804user58804

212




212




New contributor




user58804 is a new contributor to this site. Take care in asking for clarification, commenting, and answering.
Check out our Code of Conduct.





New contributor





user58804 is a new contributor to this site. Take care in asking for clarification, commenting, and answering.
Check out our Code of Conduct.






user58804 is a new contributor to this site. Take care in asking for clarification, commenting, and answering.
Check out our Code of Conduct.








  • 1




    $begingroup$
    Please don't change your question after someone has answered it. Now the answer references D, E, and F, and makes no sense. I'm editing the answer to match the revised question, but in the future it's best not to make changes that make the answers obsolete ... especially when the changes are superficial like this.
    $endgroup$
    – Rubio
    1 hour ago














  • 1




    $begingroup$
    Please don't change your question after someone has answered it. Now the answer references D, E, and F, and makes no sense. I'm editing the answer to match the revised question, but in the future it's best not to make changes that make the answers obsolete ... especially when the changes are superficial like this.
    $endgroup$
    – Rubio
    1 hour ago








1




1




$begingroup$
Please don't change your question after someone has answered it. Now the answer references D, E, and F, and makes no sense. I'm editing the answer to match the revised question, but in the future it's best not to make changes that make the answers obsolete ... especially when the changes are superficial like this.
$endgroup$
– Rubio
1 hour ago




$begingroup$
Please don't change your question after someone has answered it. Now the answer references D, E, and F, and makes no sense. I'm editing the answer to match the revised question, but in the future it's best not to make changes that make the answers obsolete ... especially when the changes are superficial like this.
$endgroup$
– Rubio
1 hour ago










1 Answer
1






active

oldest

votes


















3












$begingroup$


It's not possible to know what A is.


C says B is lying. So either C is a knave and B is a knight or C is a knight and B is a knave. Therefore there is one knave among (B,C).


Assume A is a knave. Then there are 2 knaves and A would lie about it. Therefore B is lying about what A said, so A, B are knaves and C is a knight.


Assume A is a knight. Then there is only one knave and B is lying about what A said, so A, C are knights and B is a knave.


In both scenarios B is a knave and C is a knight. A could be either a knight or a knave.







share|improve this answer











$endgroup$














    Your Answer








    StackExchange.ready(function() {
    var channelOptions = {
    tags: "".split(" "),
    id: "559"
    };
    initTagRenderer("".split(" "), "".split(" "), channelOptions);

    StackExchange.using("externalEditor", function() {
    // Have to fire editor after snippets, if snippets enabled
    if (StackExchange.settings.snippets.snippetsEnabled) {
    StackExchange.using("snippets", function() {
    createEditor();
    });
    }
    else {
    createEditor();
    }
    });

    function createEditor() {
    StackExchange.prepareEditor({
    heartbeatType: 'answer',
    autoActivateHeartbeat: false,
    convertImagesToLinks: false,
    noModals: true,
    showLowRepImageUploadWarning: true,
    reputationToPostImages: null,
    bindNavPrevention: true,
    postfix: "",
    imageUploader: {
    brandingHtml: "Powered by u003ca class="icon-imgur-white" href="https://imgur.com/"u003eu003c/au003e",
    contentPolicyHtml: "User contributions licensed under u003ca href="https://creativecommons.org/licenses/by-sa/3.0/"u003ecc by-sa 3.0 with attribution requiredu003c/au003e u003ca href="https://stackoverflow.com/legal/content-policy"u003e(content policy)u003c/au003e",
    allowUrls: true
    },
    noCode: true, onDemand: true,
    discardSelector: ".discard-answer"
    ,immediatelyShowMarkdownHelp:true
    });


    }
    });






    user58804 is a new contributor. Be nice, and check out our Code of Conduct.










    draft saved

    draft discarded


















    StackExchange.ready(
    function () {
    StackExchange.openid.initPostLogin('.new-post-login', 'https%3a%2f%2fpuzzling.stackexchange.com%2fquestions%2f82082%2fknights-and-knaves-question%23new-answer', 'question_page');
    }
    );

    Post as a guest















    Required, but never shown

























    1 Answer
    1






    active

    oldest

    votes








    1 Answer
    1






    active

    oldest

    votes









    active

    oldest

    votes






    active

    oldest

    votes









    3












    $begingroup$


    It's not possible to know what A is.


    C says B is lying. So either C is a knave and B is a knight or C is a knight and B is a knave. Therefore there is one knave among (B,C).


    Assume A is a knave. Then there are 2 knaves and A would lie about it. Therefore B is lying about what A said, so A, B are knaves and C is a knight.


    Assume A is a knight. Then there is only one knave and B is lying about what A said, so A, C are knights and B is a knave.


    In both scenarios B is a knave and C is a knight. A could be either a knight or a knave.







    share|improve this answer











    $endgroup$


















      3












      $begingroup$


      It's not possible to know what A is.


      C says B is lying. So either C is a knave and B is a knight or C is a knight and B is a knave. Therefore there is one knave among (B,C).


      Assume A is a knave. Then there are 2 knaves and A would lie about it. Therefore B is lying about what A said, so A, B are knaves and C is a knight.


      Assume A is a knight. Then there is only one knave and B is lying about what A said, so A, C are knights and B is a knave.


      In both scenarios B is a knave and C is a knight. A could be either a knight or a knave.







      share|improve this answer











      $endgroup$
















        3












        3








        3





        $begingroup$


        It's not possible to know what A is.


        C says B is lying. So either C is a knave and B is a knight or C is a knight and B is a knave. Therefore there is one knave among (B,C).


        Assume A is a knave. Then there are 2 knaves and A would lie about it. Therefore B is lying about what A said, so A, B are knaves and C is a knight.


        Assume A is a knight. Then there is only one knave and B is lying about what A said, so A, C are knights and B is a knave.


        In both scenarios B is a knave and C is a knight. A could be either a knight or a knave.







        share|improve this answer











        $endgroup$




        It's not possible to know what A is.


        C says B is lying. So either C is a knave and B is a knight or C is a knight and B is a knave. Therefore there is one knave among (B,C).


        Assume A is a knave. Then there are 2 knaves and A would lie about it. Therefore B is lying about what A said, so A, B are knaves and C is a knight.


        Assume A is a knight. Then there is only one knave and B is lying about what A said, so A, C are knights and B is a knave.


        In both scenarios B is a knave and C is a knight. A could be either a knight or a knave.








        share|improve this answer














        share|improve this answer



        share|improve this answer








        edited 1 hour ago









        Rubio

        30.8k567189




        30.8k567189










        answered 3 hours ago









        JayJay

        2,8742922




        2,8742922






















            user58804 is a new contributor. Be nice, and check out our Code of Conduct.










            draft saved

            draft discarded


















            user58804 is a new contributor. Be nice, and check out our Code of Conduct.













            user58804 is a new contributor. Be nice, and check out our Code of Conduct.












            user58804 is a new contributor. Be nice, and check out our Code of Conduct.
















            Thanks for contributing an answer to Puzzling Stack Exchange!


            • Please be sure to answer the question. Provide details and share your research!

            But avoid



            • Asking for help, clarification, or responding to other answers.

            • Making statements based on opinion; back them up with references or personal experience.


            Use MathJax to format equations. MathJax reference.


            To learn more, see our tips on writing great answers.




            draft saved


            draft discarded














            StackExchange.ready(
            function () {
            StackExchange.openid.initPostLogin('.new-post-login', 'https%3a%2f%2fpuzzling.stackexchange.com%2fquestions%2f82082%2fknights-and-knaves-question%23new-answer', 'question_page');
            }
            );

            Post as a guest















            Required, but never shown





















































            Required, but never shown














            Required, but never shown












            Required, but never shown







            Required, but never shown

































            Required, but never shown














            Required, but never shown












            Required, but never shown







            Required, but never shown







            Popular posts from this blog

            GameSpot

            connect to host localhost port 22: Connection refused

            Getting a Wifi WPA2 wifi connection